Đến nội dung

Hình ảnh

Giải phương trình hệ phương trình bằng phương pháp sử dụng bất đẳng thức.


  • Please log in to reply
Chủ đề này có 36 trả lời

#1
Yagami Raito

Yagami Raito

    Master Tetsuya

  • Thành viên
  • 1333 Bài viết

Đây là chuyên đề mình viết chào mừng Hoạt động Chào mừng 60 năm Chiến thắng Điện Biên Phủ 

Do chuyên đề còn rất sơ xài nên mình post lên đây để trao đổi thảo luận , từ đó đúc kết làm một chuyên đề hoàn chỉnh hơn rất mong được ủng hộ xin cám ơn !

(Password file download là 12345678)

 

 

Giải Phương Trình, Hệ Phương Trình Bằng Bất Đẳng Thức        

 

A.Lời nói đầu:

 

Bất  đẳng  thức  đại  số  là  một  vấn  đề  rất  được  yêu  chuộng  bởi học  sinh  yêu  toán  hiện  nay.                                                                   Ta có thể bắt gặp rất nhiều bài toán ngay dưới hình thức giải   phương trình hay giải hệ phương trình. Điều này đòi hỏi ở người giải toán phải có một trường bất đẳng thức vững vàng - một tư duy sắc bén, để từ đó có thể vận dụng một cách linh hoạt "những kinh nghiệm dày dặn của bất đẳng thức" để giải quyết ngay những bài phương trình và hệ phương trình. Bài viết sẽ tổng hợp và sưu tầm cho các bạn đọc thấy rõ sự phong phú phức tạp của các bài toán giải phương trình hệ phương trình bằng bất đẳng thức. Xin chân thành cám ơn !

 

 

 

 

B. Một số kiến thức cần lưu ý:

 

Ta cần chú ý một số bất đẳng thức căn bản quen thuộc sau: 

 

1. $|A|=|-A| \geq 0$. Dấu “$=$ xảy ra $\Leftrightarrow A=0$ 

 

2. $|A| \geq A$. Dấu bằng xảy ra khi $\Leftrightarrow A \geq 0$

 

3. $a^{2}\geq 0\forall a$. Dấu "=" có khi: $a=0$.

 

4. $|a|\geq a\forall a$. Dấu "=" có khi: $a\geq 0$.

 

5. $|a|+|b|\geq |a+b|$. Dấu "=" có khi: $ab\geq 0$.

 

6. $|a|-|b|\leq |a-b|$. Dấu "=" có khi: $\left\{\begin{matrix}ab\geq 0 & & \\ |a|\geq |b| & & \end{matrix}\right.$.

 

7. $a^{2}+b^{2}\geq 2ab$. Dấu "=" có khi: $a=b$

 

8. $(a+b)^{2}\geq 4ab\Leftrightarrow ab\leq (\frac{(a+b)}{2})^{2}$. Dấu "=" có khi: $a=-b$.

 

9. $\frac{1}{a}+\frac{1}{b}\geq \frac{4}{a+b}(a;b> 0)$. Dấu "=" có khi: $a=b$.

 

10. $\frac{a}{b}+\frac{b}{a}\geq 2(ab> 0)$. Dấu "=" có khi: $a=b$.

 

11. Bất đẳng thức Cô-si (AM-GM):

Với $n$ số thực dương: $a_{1};a_{2};...;a_{n}$

        Dạng1      $\frac{a_{1}+a_{2}+...+a_{n}}{n}\geq \sqrt[n]{a_{1}a_{2}...a_{n}}$

        Dạng 2       $a_{1}+a_{2}+...+a_{n}\geq n\sqrt[n]{a_{1}a_{2}...a_{n}}$

       Dạng 3      $(\frac{a_{1}+a_{2}+...+a_{n}}{n})^{n}\geq a_{1}a_{2}...a_{n}$

Dấu "=" có khi: $a_{1}=a_{2}=...=a_{n}$

 

 

 

 

12. Bất đẳng thức BCS (Bunhiakovsky):

 

Với 2 bộ số thực bất kì: ($a_{1};a_{2};...;a_{n}$);($b_{1};b_{2};...;b_{n}$):
 Dạng 1: $(a_{1}b_{1}+a_{2}b_{2}+...+a_{n}b_{n})^{2}\leq (a_{1}^{2}+a_{2}^{2}+...+a_{n}^{2})(b_{1}^{2}+b_{2}^{2}+...+b_{n}^{2})$

Dạng 2: $|a_{1}b_{1}+a_{2}b_{2}+...+a_{n}b_{n}|\leq \sqrt{a_{1}^{2}+a_{2}^{2}+...+a_{n}^{2})(b_{1}^{2}+b_{2}^{2}+...+b_{n}^{2}}$

Dấu "=" có khi: $\frac{a_{1}}{b_{1}}=\frac{a_{2}}{b_{2}}=...=\frac{a_{n}}{b_{n}}$

Dạng 3: $a_{1}b_{1}+a_{2}b_{2}+...+a_{n}b_{n}\leq \sqrt{(a_{1}^{2}+a_{2}^{2}+...+a_{n}^{2})(b_{1}^{2}+b_{2}^{2}+...+b_{n}^{2})}$

Dấu "=" có khI: $\frac{a_{1}}{b_{1}}=\frac{a_{2}}{b_{2}}=...=\frac{a_{n}}{b_{n}}>0$

13. Bất đẳng thức BCS dạng cộng mẫu: (Cauchy-Swarchz)

Với $\forall x_{i}>0;i=\overline{1,n}$ ta có:

$\frac{a_{1}^{2}}{x_{1}}+\frac{a_{2}^{2}}{x_{2}}+...+\frac{a_{n}^{2}}{x_{n}}\geq \frac{(a_{1}+a_{2}+...+a_{n})^{2}}{x_{1}+x_{2}+...+x_{n}}$

Chứng minh: Xét $(a_{1}+a_{2}+...+a_{n})^{2}=(\frac{a_{1}}{\sqrt{x_{1}}}.\sqrt{x_{1}}+\frac{a_{2}}{\sqrt{x_{2}}}.\sqrt{x_{2}}+...+\frac{a_{n}}{\sqrt{x_{n}}}.\sqrt{x_{n}})^{2}\leq (\frac{a_{1}^{2}}{x_{1}}+\frac{a_{2}^{2}}{x_{2}}+...+\frac{a_{n}^{2}}{x_{n}})(x_{1}+x_{2}+...+x_{n})$ (Áp dụng BCS)
Vậy bất đẳng thức được chứng minh.

 

14. Bất đẳng thức Minkopsky:

Cho 2 dãy số thực dương: $(a_{1};a_{2};...;a_{n});(b_{1};b_{2};...;b_{n})$ ta có:

$\sqrt{a_{1}^{2}+b_{1}^{2}}+\sqrt{a_{2}^{2}+b_{2}^{2}}+...+\sqrt{a_{n}^{2}+b_{n}^{2}}\geq \sqrt{(a_{1}+a_{2}+...+a_{n})^{2}+(b_{1}+b_{2}+...+b_{n})^{2}}$

Dấu "=" xảy ra khi: $\frac{a_{1}}{b_{1}}=\frac{a_{2}}{b_{2}}=...=\frac{a_{n}}{b_{n}}$.

 

15. Lũy thừa với số mũ hữu tỉ: 

Cho số thực $a>0$, $\frac{m}{n}$ là một số hữu tỉ, trong đó  $m,n\in \mathbb{N^*},n>1$

Thì lũy thừa của $a$ với số mũ $\dfrac{m}{n}$ là  $a^{\frac{m}{n}}=\sqrt[n]{a^{m}}$

 

 

C. Các bài toán giải phương trình bằng bất đẳng thức.

 

Ta cùng đến với một số bài  toán lý thú sau : 

 

 

Bài toán 1       Giải phương trình

$2(\frac{{{x}^{10}}}{{{y}^{2}}}+\frac{{{y}^{10}}}{{{x}^{2}}})+{{x}^{16}}+{{y}^{16}}=4{{(1+{{x}^{2}}{{y}^{2}})}^{2}}-10$

 

Lời giải :  Điều kiện: $x,y \neq 0$

 

Áp dung  bất đẳng thức cauchy cho 4 số dương ta có:

$$\dfrac{x^{10}}{y^2}+\dfrac{y^{10}}{x^2}+1+1 \geq 4.\sqrt[4]{\dfrac{x^{10}.y^{10}}{x^2.y^2}}=4x^2y^2$$

$$\Rightarrow 2(\frac{{{x}^{10}}}{{{y}^{2}}}+\frac{{{y}^{10}}}{{{x}^{2}}}+2)\ge 8{{x}^{2}}{{y}^{2}}$$

 

Và:                             $${{x}^{16}}+{{y}^{16}}+1+1\ge 4.\sqrt[4]{{{x}^{16}}.{{y}^{16}}}=4{{x}^{4}}{{y}^{4}}$$

$$\Rightarrow 2(\frac{{{x}^{10}}}{{{y}^{2}}}+\frac{{{y}^{10}}}{{{x}^{2}}}+2)+{{x}^{16}}+{{y}^{16}}+2\ge 4{{x}^{4}}{{y}^{4}}+8{{x}^{2}}{{y}^{2}}$$

$$\Rightarrow 2(\frac{{{x}^{10}}}{{{y}^{2}}}+\frac{{{y}^{10}}}{{{x}^{2}}})+{{x}^{16}}+{{y}^{16}}\ge 4{{(1+{{x}^{2}}{{y}^{2}})}^{2}}-10$$

 

Dấu đẳng thức xảy ra khi và chỉ khi:

$x^2=y^2=1 \Leftrightarrow |x|=|y|=1$

 

Vậy phương trình  đã cho có các nghiệm $(x;y)$ là :

$(1;1),(-1;-1),(1;-1),(-1;1)$

 

 

Bài toán 2     Giải phương trình

$x^4+4=2\sqrt{x^4+4}+2\sqrt{x^4-4}$

 

Lời giải

Áp dụng bất đẳng thức cauchy cho hai số không âm ta có: 

$$x^4+4\geq 4x^2$$ (1)

 

Theo BĐT cauchy-schwarz ta có:

$$(a+b) \leq 2(a^2+b^2)$ $\Leftrightarrow a+b \leq \sqrt{2(a^2+b^2)}$$

 

Dấu "=" xảy ra khi $a=b \geq 0$

 

Trở lại bài toán, áp dụng BĐT trên ta có :

$$2\sqrt{{{x}^{4}}+4}+2\sqrt{{{x}^{4}}-4}\le \sqrt{2[4({{x}^{4}}+4)+4({{x}^{4}}-4)]}\Leftrightarrow 2\sqrt{{{x}^{4}}+4}+2\sqrt{{{x}^{4}}-4}\le 4{{x}^{2}}$$ (2)

 

Từ $(1);(2)$ ta có dấu bằng xảy ra khi: 

$$\left\{\begin{matrix} x^4=4 & & \\ 2\sqrt{x^4+4}=2\sqrt{x^4-4} & & \end{matrix}\right. \Leftrightarrow x \in \oslash$$

Vậy phương trình đã cho vô nghiệm.

 

 

 

Bài toán 3         Giải phương trình:

$2\sqrt[4]{27x^2+24x+\dfrac{28}{3}}=1+\sqrt{\dfrac{27}{2}x+6}$

 

Lời giải

 

Ta có: $2\sqrt[4]{27{{x}^{2}}+24x+\frac{28}{3}}=1+\sqrt{\frac{3(9x+4)}{2}}\text{ }\Leftrightarrow \sqrt[4]{\frac{{{(9x+4)}^{2}}}{3}+4}=1+\sqrt{\frac{3(9x+4)}{2}}(1)$

 

 

Điều kiện: $9x+4=y \geq 0$. Khi đó $(1)$ trở thành: 

$2\sqrt[4]{\dfrac{y^2}{3}+4}=1+\sqrt{\dfrac{3y}{2}}\Leftrightarrow 4\sqrt{\dfrac{y^2}{3}+4}=1+\dfrac{3y}{2}+\sqrt{6y}$

 

Áp dụng bất đẳng thức cauchy ta có 

$\sqrt{6y} \leq \dfrac{6+y}{2}$        $\Rightarrow 4\sqrt{\dfrac{y^2}{3}+4} \leq 2y+4$

$\Leftrightarrow 4(\dfrac{y^3}{3}+4)\leq (y+2)^2\Leftrightarrow \dfrac{(y-6)^2}{3} \leq 0$ 

 

Ta lại có: $(y-6)^2 \geq 0$ nên $y=6$

 

Từ đó $x=\dfrac{y-4}{9}=\dfrac{2}{9}$ thỏa mãn điều kiện ban đầu. 

 

Vậy phương trình đã cho có nghiệm duy nhất $x=\dfrac{2}{9}$

 

 

 

Bài toán 4        Giải phương trình 

$13\sqrt{x^2-x^4}+9\sqrt{x^2+x^4}=16$

 

Lời giải     

 Áp dụng bất đẳng thức cauchy ta có : 

$\sqrt{4(1-x^2)x^2} \leq \dfrac{4(1-x^2)+x^2}{2}=\dfrac{4-3x^2}{2}$

$\Rightarrow 13\sqrt{x^2-x^4} \leq \dfrac{52-39x^2}{4} (1)$

Tương tự:    $\sqrt{9x^2.4(1+x^2)} \leq \dfrac{13x^2+4}{2}$

$\Rightarrow 9\sqrt{x^2+x^4} \leq \dfrac{39x^2+12}{4} (2)$

 

Cộng vế theo vế của $(1)$ và $(2)$ ta có $13\sqrt{x^2-x^4}+9\sqrt{x^2+x^4} \leq 16$

 

Dấu bằng xảy ra khi $\left\{\begin{matrix} 4(1-x^2)=x^2 & & \\ 9x^2=4(1+x^2) & & \end{matrix}\right.\Leftrightarrow x=\frac{2\sqrt{5}}{5}$

 

Nhận xét: bạn đọc có thể giải bài toán trên bằng phương pháp dùng BĐT Cauchy schwarzt:

$VT^2 \leq (13+27)[13(x^2-x^4)+3(x^2+x^4)]=80(8x^2-5x^4)=80[\dfrac{16}{5}-5(x^2-\dfrac{4}{5})^2] \leq 16^2$......

 

 

 

Bài toán 5      Giải phương trình 

$\sqrt{x^2+2x}+\sqrt{2x-1}=\sqrt{3x^2+4x+1}$

 

Lời giải Tập xác định 

$x^2+2x \geq 0\Leftrightarrow \begin{bmatrix} x\geq 0 & & \\ x \leq -2 & & \end{bmatrix} \\2x-1 \geq \Leftrightarrow x \geq \dfrac{1}{2} \\3x^2+4x+1 \geq 0 \Leftrightarrow \begin{bmatrix} x \leq -1 & & \\ x \geq \dfrac{1}{3} & & \end{bmatrix}$

Ta có tập xác định là {$x \in \mathbb{R}|x\geq \dfrac{1}{2}$}

 

Áp dụng bất đẳng thức Cauchy schwarz ta có:

$\sqrt{x}.\sqrt{x+2}+1.\sqrt{2x-1}\le \sqrt{\sqrt{{{x}^{2}}}+1}.\sqrt{\sqrt{{{(x+2)}^{2}}}.\sqrt{{{(2x-1)}^{2}}}}=\sqrt{(x+1)(x+2+2x-1)}=\sqrt{3{{x}^{2}}+4x+1}$

 

Đẳng thức xảy ra khi và chỉ khi $\frac{\sqrt{x}}{\sqrt{x+2}}=\frac{1}{\sqrt{2x-1}}\Leftrightarrow \sqrt{2{{x}^{2}}-x}=\sqrt{x+2}\Leftrightarrow {{x}^{2}}-x+1=0\text{ }\Leftrightarrow x=\frac{1\pm \sqrt{5}}{2}\Leftrightarrow x=\frac{1+\sqrt{5}}{2}$

(do $x \geq \dfrac{1}{2}$)

 

Vậy phương trình có nghiệm là $x=\dfrac{1+\sqrt{5}}{2}$

 

 

D. Các bài toán giải hệ phương trình bằng bất đẳng thức.

 

Như các bạn đọc đã biết về phần này, việc sử dụng bất đẳng thức trong giải hệ phương trình thì rất phong phú,rất đa dạng và đầy đặc sắc tinh tế, sau đây mình xin giới thiệu tới bạn đọc một số bài toán sưu tầm hay và phổ biến:

 

Bài toán 1 Giải hệ phương trình:  

$\left\{\begin{matrix} x^2+y^2=1 & & \\ 125y^5-125y^3+6\sqrt{15}=0 & & \end{matrix}\right.$

 

Lời giải

 

Hệ phương trình đã cho tương đương với: $\left\{\begin{matrix} x^2+y^2=1 & & \\ y^3(1-y^2)=\dfrac{6\sqrt{15}}{125} & & \end{matrix}\right.$

$\Leftrightarrow \left\{\begin{matrix} x^2=1-y^2 & & \\ y^3x^2=\dfrac{6\sqrt{15}}{125} & & \end{matrix}\right.\Leftrightarrow \left\{\begin{matrix} x^2=1-y^2(1) & & & \\ y>0 & & & \\ y^6x^4=\dfrac{4.3^3}{5^5}(2) & & & \end{matrix}\right.$

 

Áp dụng bất đẳng thức cauchy cho 5 số không âm $y^2,y^2,y^2,\dfrac{3}{2}x^2,\dfrac{3}{2}x^2$ ta có:

$5\sqrt[5]{y^6.\frac{9}{4}x^4} \leq 3(x^2+y^2)=3\Leftrightarrow y^6x^4 \leq \dfrac{4.3^2}{5^5}(3)$

 

Kết hợp $(2)$ và $(3)$ thì bất đẳng thức $(3)$ trở thành đẳng thức.Lúc đó $y^2=\dfrac{3}{2}x^2$

 

Kết hợp với $(1)$ ta có: 

$x^2+\dfrac{3}{2}x^2=1$ $\Leftrightarrow x^2=\frac{2}{5}\Leftrightarrow x=\pm \frac{\sqrt{10}}{5}$

 

Do đó $y^2=\dfrac{3}{5}$$\Leftrightarrow y=\frac{\sqrt{15}}{5}$(vì $y>0$)

 

Vậy hệ phương trình có hai nghiệm $(x;y)$ là $(\frac{\sqrt{10}}{5};\frac{\sqrt{15}}{5})$ và $(\dfrac{-\sqrt{10}}{5};\dfrac{\sqrt{15}}{5})$

 

Nhận xét: Việc sử dụng bất đẳng thức trong bài toán trên thật tinh tế, ngoài ra bạn đọc có thể giải bài toán trên bằng cách đặt $t=y.\dfrac{\sqrt{15}}{5}$ ($t>0$)

Khi đó $3t^5-5t^3+2=0$ $\Leftrightarrow (t-1)(3t^3+6t^2+4t+2)=0$.....mời bạn đọc giải tiếp và tìm kiếm thêm nhiều lời giải hay khác

 

 

 Bài toán 2 Giải hệ phương trình: 

$\left\{\begin{matrix} \sqrt{x}+\sqrt[4]{32-x}-y^2=-3 & & \\ \sqrt[4]{x}+\sqrt{32-x}+6y=24 & & \end{matrix}\right.$

Lời giải   Điều kiện $0 \leq x \leq 32$

 

Cộng vế theo vế ta có : 

$(\sqrt{x}+\sqrt{32-x})+(\sqrt{\sqrt[4]{x}}+\sqrt[4]{32-x})={{y}^{2}}-6y+21.$

$y^2-6y+21=(y-3)^2+12 \geq 12$ với mọi  và đẳng thức xảy ra khi $y=3(1)$

 

Mặt khác, áp dụng bất đẳng thức Cauchy Schwarz ta có

$(\sqrt{x}+\sqrt{32-x})^2 \leq (1^2+1^2)(x+32-x)=64$

$\Rightarrow \sqrt{x}+\sqrt{32-x} \leq 8$ với mọi $x \in [0;32]$

và đẳng thức xảy ra khi và chỉ khi $\sqrt{x}=\sqrt{32-x} \Leftrightarrow x=16(2)$

 

Lại áp dụng bất đẳng thức Cauchy schwarz và kết hợp $(2)$ ta có 

$(\sqrt[4]{x}+\sqrt[4]{32-x})^2 \leq (1^2+1^2)(\sqrt{x}+\sqrt{32-x}) \leq 16$

$\Rightarrow \sqrt[4]{x}+\sqrt[4]{32-x} \leq 4$ với mọi $x \in [0;32]$

và đẳng thức xảy ra khi và chỉ khi $\sqrt[4]{x}=\sqrt[4]{32-x} \Leftrightarrow x=16 (3)$

 

 

Từ $(2),(3)$ suy ra 

$(\sqrt{x}+\sqrt{32-x})+(\sqrt{\sqrt[4]{x}}+\sqrt[4]{32-x}) \leq 12 (4)$

 

Từ $(1),(4)$ ta có dấu bằng xảy ra khi $x=16$ và $y=3$

 

Thử vào hệ đã cho ta thấy thỏa mãn.

 

Vậy hệ có nghiệm duy nhất $(x;y)=(16;3)$

Bài toán 3    Giải hệ phương trình:           $\left\{\begin{matrix}\frac{x}{\sqrt{y}}+\frac{y}{\sqrt{x}}=xy & & \\ x^{2010}+y^{2010}=8\sqrt{(xy)^{2007}} & & \end{matrix}\right.$

Lời giải    Điều kiện là $x,y >0$

Áp dụng bất đẳng thức Cauchy ta có:

$xy=\dfrac{x}{\sqrt{y}}+\dfrac{y}{\sqrt{x}}\geq 2\sqrt[4]{xy}\Rightarrow xy\geq \sqrt[3]{16}\;\;(1)$

Và :              $8\sqrt{(xy)^{2007}}=x^{2010}+y^{2010}\geq 2\sqrt{(xy)^{2010}}\Rightarrow xy\leq \sqrt[3]{16}\;\;(2)$

Từ $(1)(2)$ có $xy=\sqrt[3]{16}$. Đẳng thức xảy ra khi $x=y=\sqrt[6]{16}$. Đó là nghiệm của hệ phương trình.

 

 

 

Bài toán 4          Giải  hệ phương trình

\[\left\{\begin{matrix} (x+7y)\sqrt{x}+(y+7x)\sqrt{y}=8\sqrt{2xy(x+y)}\\ 2(1-y)\sqrt{x^2+2x-1}=y^2-2x-1 \end{matrix}\right.\]

 

Lời giải

Đặt $a=\sqrt{x},b=\sqrt{y}$($a;b \geq 0$)

 

Khi đó phương trình (1) trở thành:

$(a+b)(a^2+6ab+b^2)=4.\sqrt{ab}.2\sqrt{2ab(a^2+b^2)}$

 

Áp dụng bất đẳng thức Cauchy cho 2 số không âm ta có :

$2\sqrt{2ab(a^2+b^2)} \leq (a+b)^2$

Suy ra

$(a^2+6ab+b^2) \leq 4(a+b)\sqrt{ab}$ (3)

Chia hai vế cho $b^2$ và đặt $\frac{\sqrt{a}}{\sqrt{b}}=t$  ($t \geq 0$) thì ta có (3) trở thành

$t^4-4t^3+6t^2-4t+1 \leq 0$

Tiếp tục chia hai vế cho $t^2$ và đặt $t+\frac{1}{t}=k$ ($k \geq 0$) thì ta có:

$k^2-4k+4 \leq 0$.

                                   \[\Leftrightarrow (k-2)^2 \leq 0\]

Do đó $k=2$ nên $t=1$ suy ra $x=y$

Thay vào phương trình thứ (2) ta có phương trình:

$2(1-x)\sqrt{x^2+2x-1}=x^2-2x-1$

Giải phương trình trên xin dành cho bạn đọc !

 

 

 

 

Bài toán 5         Giải hệ phương trình :

$\left\{\begin{matrix} xyz=1 & & \\ x^3+y^3+z^3=x+y+z & & \end{matrix}\right.$(x,y,z>0)

 

 

Lời giải

Cách 1:

Với $x,y,z>0$ Áp dụng bất đẳng thức Cauchy ta có

$$x^3+y^3+z^3 \geq 3\sqrt[3]{xyz}=3$$

$\Rightarrow 2(x^3+y^3+z^3) \geq 6 \Rightarrow 3(x^3+y^3+z^3) \geq x^3+y^3+z^3+6$

 

Tiếp tục áp dụng bất đẳng thức Cauchy ta có :

$$x^3+1+1 \geq 3x \Rightarrow x^3+2 \geq 3x$$

 

Tương tự : $y^3+2 \geq 3$ ; $z^3+2 \geq 3x$

$\Rightarrow x^3+y^3+z^3 \geq x+y+z$$

Dấu bằng xảy ra khi $x=y=z=1$.

 

Vậy hệ phương trình có một nghiệm $(1;1;1)$

 

Cách 2:

Ta có  :                $x^3+y^3+z^3-3xyz=(x+y+z)(x^2+y^2+z^2-xy-yz-zx)$

$=\dfrac{1}{2}(x+y+z)[(x-y)^2+(y-z)^2+(z-x)^2] \geq 0$

$\Rightarrow x^3+y^3+z^3 \geq 3xyz=3$ (1)

 

Ta lai có: $(x-1)^2\geq 0 \Rightarrow (x-1)^2(x+2) \geq 0$

$\Rightarrow x^3+2 \geq 3x$

 

Lập luận tương tự ta có: $y^3+2 \geq 3y$; $z^3+2 \geq 3z$

 

Suy ra

$x^3+y^3+z^3 +6 \geq 3(x+y+z)$ (2)

 

Từ (1) và (2) ta có $x^3+y^3+z^3 \geq x+y+z$

 

Đẳng thức xảy ra khi $x=y=z=1$

 

Vậy hệ phương trình có nghiệm duy nhất $(x,y,z)=(1;1;1)$

 

File gửi kèm


:nav: Học gõ công thức toán học tại đây

:nav: Hướng dẫn đặt tiêu đề tại đây

:nav: Hướng dẫn Vẽ hình trên diễn đàn toán tại đây

--------------------------------------------------------------

 


#2
Yagami Raito

Yagami Raito

    Master Tetsuya

  • Thành viên
  • 1333 Bài viết

 Bài tập 

Giải các Phương trình sau:

 

 1.                 $20^{x^2+3}+11^{y^2+2}+2006^{x^2+1}=10127$

 

2.             $\sqrt{-x^2+3x+4}+\sqrt{-y^2+2y+2}=\sqrt{-x^2+5x+14}$

 

3.              $\dfrac{2005}{x+y}+\dfrac{x}{y+2004}+\dfrac{y}{4009}+\dfrac{2004}{2005+x}=\dfrac{2}{z}$($x,y,z\in \mathbb{N*}$ )

 

4.               $\sqrt{-4x^4y^2+16x^2y+9}-\sqrt{y^2x^2-2y^2}=2(x^2+\dfrac{1}{x^2})$, với $x>0$

 

5.               $13[(x^2-3x+6)^2+(x^2-2x+7)^2]=(5x^2-12+33)^2$

 

6.                  $\sqrt{x^2+x+1}-\sqrt{x^2-x+1}=4x^2-4+\dfrac{32}{x^2(2x^2+3)^2}$

 

7.                          $\dfrac{11}{x^2}-\dfrac{25}{(x+5)^2}=1$

 

8.              $\dfrac{(x^2+16|x|+48)(x^2+12|x|+27)}{x^2}=675$

 

9.              $\sqrt{x^2-x+19}+\sqrt{7x^2+8x+13}+\sqrt{13x^2+17x+7}=3\sqrt{3}(x+2)$

 

10.                        $32x^4+(4x-1)^4=\dfrac{1}{27}$

(ở bài toán trên chú ý bài toán phụ $a^4+b^4+c^4=\dfrac{1}{27}(a+b+c)^4 \Leftrightarrow a=b=c$)


Bài viết đã được chỉnh sửa nội dung bởi Yagami Raito: 02-05-2014 - 17:06

:nav: Học gõ công thức toán học tại đây

:nav: Hướng dẫn đặt tiêu đề tại đây

:nav: Hướng dẫn Vẽ hình trên diễn đàn toán tại đây

--------------------------------------------------------------

 


#3
einstein627

einstein627

    Trung sĩ

  • Thành viên
  • 102 Bài viết
 

9.              $\sqrt{x^2-x+19}+\sqrt{7x^2+8x+13}+\sqrt{13x^2+17x+7}=3\sqrt{3}(x+2)$

 

9.(chưa đọc xong chuyên đề nhưng thôi thấy bài nào làm được thì làm )

$\sqrt{x^{2}-x+19}\doteq \sqrt{(x-\frac{1}{2})^{2}+18\frac{3}{4}}$$\geq 18\frac{3}{4}$

$7x^{2}+8x+13=(2x-1)^{2}+3(x+2)^{2}\geq 3(x+2)^{2}$

$13x^{2}+17x+7= \frac{(2x-1)^{2}}{4}+\frac{3(4x+3)^{2}}{4}\geq 3\frac{(4x+3^{2})}{4}$

​Thay vào pt ban đầu ta có VP$\geq$ VT nen VT=VP $\Leftrightarrow x=\frac{1}{2}$


Bài viết đã được chỉnh sửa nội dung bởi einstein627: 27-04-2014 - 19:14

-Học từ ngày hôm qua, sống ngày hôm nay, hi vọng cho ngày mai. Điều quan trọng nhất là không ngừng đặt câu hỏi.

-Albert Einstein

 
-Khi Bạn Sắp Bỏ Cuộc, Hãy Nhớ Tới Lý Do Khiến Bạn Bắt Đầu.

 


#4
Viet Hoang 99

Viet Hoang 99

    $\textbf{Trương Việt Hoàng}$

  • Điều hành viên THPT
  • 2291 Bài viết

 Bài tập 

Giải các Phương trình sau:

 

10.                        $32x^4+(4x-1)^4=\dfrac{1}{27}$

(ở bài toán trên chú ý bài toán phụ $a^4+b^4+c^4=\dfrac{1}{27}(a+b+c)^4 \Leftrightarrow a=b=c$)

Hế mấy BĐT đầu cóp của tui à  :luoi:

$10/$

Chú ý thì chú ý :D

BĐT phụ: $\sum \frac{a^4}{1}\geq \frac{(a^2+b^2+c^2)^2}{3}\geq \frac{\left[\frac{(a+b+c)^2}{3}\right]^2}{3}=\frac{(a+b+c)^4}{27}$

+ Bước 1 là BCS dạng cộng mẫu

+ Bước 2 là BCS

Vậy $a^4+b^4+c^4\geq \dfrac{1}{27}(a+b+c)^4$

 

Có:

$PT\Leftrightarrow 32x^4+(4x-1)^4=\dfrac{1}{27}\Leftrightarrow 16x^4+16x^4+(4x-1)^4=\frac{1}{27}$

Áp dụng BĐT phụ trên thì

$VT=16x^4+16x^4+(4x-1)^4\geq \frac{(2x+2x+1-4x)^4}{27}=\frac{1}{27}=VP$

Dấu = có khi: $2x=1-4x\Leftrightarrow x=\frac{1}{6}$


Bài viết đã được chỉnh sửa nội dung bởi Viet Hoang 99: 28-06-2014 - 16:54


#5
henry0905

henry0905

    Trung úy

  • Thành viên
  • 892 Bài viết

Mình có một góp ý nhỏ ở bài $13\sqrt{x^{2}-x^{4}}+9\sqrt{x^{2}+x^{4}}=16$

Bài này không dễ gì tìm được hệ số cân bằng (Mình nghe được nó đã từng xuất hiện trong kì thi Olympic 30-4 và chỉ một bạn giải được).

Dạng tổng quát: $ax\sqrt{1+x^{2}}+bx\sqrt{1-x^{2}}$

$ax\sqrt{1+x^{2}}+bx\sqrt{1-x^{2}}=\frac{a}{s}.xs\sqrt{1+x^{2}}+\frac{b}{t}.tx\sqrt{1-x^{2}}\leq \frac{1}{2}(as+\frac{a}{s}+bt-\frac{b}{t})x^{2}+\frac{1}{2}(\frac{a}{s}+\frac{b}{t})$

Dấu bằng xảy ra khi $\left\{\begin{matrix} s^{2}x^{2}=1+x^{2} & \\ t^{2}x^{2}=1-x^{2} & \end{matrix}\right. \Leftrightarrow s^{2}-t^{2}=2$

Vậy để tìm được $s,t$ ta lập hệ phương trình:

$\left\{\begin{matrix} s^{2}-t^{2}=2 & \\ as+\frac{a}{s}+bt-\frac{b}{t}=0 & \end{matrix}\right.$

Nguồn: sưu tầm.



#6
Viet Hoang 99

Viet Hoang 99

    $\textbf{Trương Việt Hoàng}$

  • Điều hành viên THPT
  • 2291 Bài viết

Mình có một góp ý nhỏ ở bài $13\sqrt{x^{2}-x^{4}}+9\sqrt{x^{2}+x^{4}}=16$

Bài này không dễ gì tìm được hệ số cân bằng (Mình nghe được nó đã từng xuất hiện trong kì thi Olympic 30-4 và chỉ một bạn giải được).

Dạng tổng quát: $ax\sqrt{1+x^{2}}+bx\sqrt{1-x^{2}}$

$ax\sqrt{1+x^{2}}+bx\sqrt{1-x^{2}}=\frac{a}{s}.xs\sqrt{1+x^{2}}+\frac{b}{t}.tx\sqrt{1-x^{2}}\leq \frac{1}{2}(as+\frac{a}{s}+bt-\frac{b}{t})x^{2}+\frac{1}{2}(\frac{a}{s}+\frac{b}{t})$

Dấu bằng xảy ra khi $\left\{\begin{matrix} s^{2}x^{2}=1+x^{2} & \\ t^{2}x^{2}=1-x^{2} & \end{matrix}\right. \Leftrightarrow s^{2}-t^{2}=2$

Vậy để tìm được $s,t$ ta lập hệ phương trình:

$\left\{\begin{matrix} s^{2}-t^{2}=2 & \\ as+\frac{a}{s}+bt-\frac{b}{t}=0 & \end{matrix}\right.$

Nguồn: sưu tầm.

Anh nên nói phần $C$ - Bài toán $4$  :luoi:

 

 

 Bài tập 

Giải các Phương trình sau:

 

 

 

8.              $\dfrac{(x^2+16|x|+48)(x^2+12|x|+27)}{x^2}=675$

 

$8/$ 
Đk: $x\neq 0$

 

Biến đổi: $A=\frac{(x^2+16\left | x \right |+48)(x^2+12\left | x \right |+27)}{x^2} \\ =\frac{(\left | x \right |+4)(\left | x \right |+12)(\left | x \right |+3)(\left | x \right |+9)}{\left | x \right | ^2} \\  =\frac{(\left | x \right | ^2+13\left | x \right |+36)(\left | x \right | ^2+15\left | x \right |+36)}{\left | x \right | ^2} \\  =\Big( \left | x \right |+\frac{36}{\left | x \right |}+13 \Big) \Big( \left | x \right |+\frac{36}{\left | x \right |}+15 \Big)$

 

Có $\left | x \right | + \frac{36}{\left | x \right |} \geq 12$ (Cauchy) nên $A \geq (12+13)(12+15)=675$

 

Dấu = khi $\left | x \right |=\frac{36}{\left | x \right |}$ hay $x =\pm 6$


Bài viết đã được chỉnh sửa nội dung bởi Viet Hoang 99: 28-06-2014 - 16:55


#7
Kaito Kuroba

Kaito Kuroba

    Thiếu úy

  • Thành viên
  • 656 Bài viết

 

 1.                 $20^{x^2+3}+11^{y^2+2}+2006^{x^2+1}=10127$

1.

vì $x^2,y^2\geq 0$

nên ta được: $$20^{x^2+3}+11^{y^2+2}+2006^{x^2+1}\geq 20^{3}+11^{2}+2006^{1}=10127; "="\Leftrightarrow x=y=0$$



#8
Kaito Kuroba

Kaito Kuroba

    Thiếu úy

  • Thành viên
  • 656 Bài viết

 


7.                          $\dfrac{11}{x^2}-\dfrac{25}{(x+5)^2}=1$

7.

$$pt\Rightarrow 11(x+5)^5-25x^2=x^2(x+5)^2$$
$$\Leftrightarrow 11(x^2+10x+25)-25x^2-x^2(x^2+10x+25)=0$$
$$\Leftrightarrow x^4+10x^3+39x^2-110x-275=0$$
$$\Leftrightarrow (x^2+11x+55)(x^2-x-5)=0$$

 

P/s: bài này dùng bdt thì chưa suy nghĩ ra!!!! :lol:



#9
Yagami Raito

Yagami Raito

    Master Tetsuya

  • Thành viên
  • 1333 Bài viết

7.

$$pt\Rightarrow 11(x+5)^5-25x^2=x^2(x+5)^2$$
$$\Leftrightarrow 11(x^2+10x+25)-25x^2-x^2(x^2+10x+25)=0$$
$$\Leftrightarrow x^4+10x^3+39x^2-110x-275=0$$
$$\Leftrightarrow (x^2+11x+55)(x^2-x-5)=0$$

 

P/s: bài này dùng bdt thì chưa suy nghĩ ra!!!! :lol:

 

$\boxed{7}$ Cách 2 : Giải sử dụng BĐT :D :D 

$\dfrac{11}{x^2}-\dfrac{25}{(x+5)^3}=1$

Đk:$x \neq -5$ và $x \neq 0$

Đặt $x+5=y$ ta có $x^2=(y-5)^2=y^2-10y+25$

Phương trình đã cho trở thành: $y^4-10y^3+39y^2-250y+625=0$

Vì $y \neq 0$ nên: 

$(y^2+\dfrac{625}{y^2})-10(y+\dfrac{25}{y})+38=0$ (1)

Đặt $z=y+\dfrac{25}{y}\Rightarrow |z|=|y+\dfrac{25}{y}|=|y|+\dfrac{25}{|y|} \geq 10$

$(1)$ trở thành $z^2-10z-11=0$

$\Rightarrow z=11$ d0 $|z| \geq 10$

Thay $z=11$ vào $z=y+\dfrac{25}{y}$

Ta được $y =\dfrac{11 \pm \sqrt{21}}{2}$

Từ đó: $x=\dfrac{1\pm \sqrt{21}}{2}$ là nghiệm của phương trình !


Bài viết đã được chỉnh sửa nội dung bởi Yagami Raito: 28-04-2014 - 10:03

:nav: Học gõ công thức toán học tại đây

:nav: Hướng dẫn đặt tiêu đề tại đây

:nav: Hướng dẫn Vẽ hình trên diễn đàn toán tại đây

--------------------------------------------------------------

 


#10
Yagami Raito

Yagami Raito

    Master Tetsuya

  • Thành viên
  • 1333 Bài viết

 Bài tập 

Giải các Phương trình sau:

 

 

 

5.               $13[(x^2-3x+6)^2+(x^2-2x+7)^2]=(5x^2-12+33)^2$

 

 

$\boxed{5}$ Áp dụng BĐT cauchy-schwart ta có: 

$(2^2+3^2)[(x^2-3x+6)^2+(x^2-2x+7)^2] \geq [2(x^2-3x+6)+3(x^2-2x+7)]^2=(5x^2-12x+33)^2$

Dấu $=$ xảy ra khi $3(x^2-3x+6)=2(x^2-2x+7) \Leftrightarrow x^2-5x+4=0$

Vậy phương trình có 2 nghiệm $x=1$;$x=4$

 

$boxed{3}$ Ai làm câu này xem  (-_-)!!


:nav: Học gõ công thức toán học tại đây

:nav: Hướng dẫn đặt tiêu đề tại đây

:nav: Hướng dẫn Vẽ hình trên diễn đàn toán tại đây

--------------------------------------------------------------

 


#11
buiminhhieu

buiminhhieu

    Thượng úy

  • Thành viên
  • 1150 Bài viết

Bài tập tiếp theo:

Giải các PT sau:

11.$\sqrt{4x-1}+\sqrt{4x^{2}-1}=1$

12.$\sqrt{3x^{2}+6x+7}+\sqrt{5x^{2}+10x+14}=4-2x-x^{2}$

13.$\sqrt{x^{2}-x+19}+\sqrt{7x^{2}+8x+13}+\sqrt{13x^{2}+17x+17}=3\sqrt{3}(x+2)$

14.$\frac{x-3x^{2}}{2}+\sqrt{2x^{4}-x^{3}+7x^{2}-3x+3}=2$

15.$\sqrt{2-x^{2}}+\sqrt{2-\frac{1}{x^{2}}}=4-(x+\frac{1}{x})$

16.$\frac{2\sqrt{2}}{\sqrt{x+1}}+\sqrt{x}=\sqrt{x+4}$

17.$3\sqrt{x^{2}-2}=\sqrt{2-x^{3}}$

18.$2x^{2}-11x+21-3\sqrt[3]{4x-4}=0$

19.$x\sqrt{y-1}+2y\sqrt{x-1}=\frac{3xy}{2}$

20.$\sqrt{(x+2)(2x-1)}-3\sqrt{x+6}=4-\sqrt{(x+6)(2x-1)}+3\sqrt{x+2}$

p/s:Làm  cùng Hiếu A


Bài viết đã được chỉnh sửa nội dung bởi buiminhhieu: 02-05-2014 - 15:35

%%- Chuyên Vĩnh Phúc

6cool_what.gif


#12
Kaito Kuroba

Kaito Kuroba

    Thiếu úy

  • Thành viên
  • 656 Bài viết

11.$\sqrt{4x-1}+\sqrt{4x^{2}-1}=1$

11.

ĐK:$x\geq \frac{1}{2}$

với Đk trên ta có:$$\sqrt{4x-1}+\sqrt{4x^2-1}\geq 1$$



#13
Kaito Kuroba

Kaito Kuroba

    Thiếu úy

  • Thành viên
  • 656 Bài viết

12.$\sqrt{3x^{2}+6x+7}+\sqrt{5x^{2}+10x+14}=4-2x-x^{2}$

12.

ta có:$$\left\{\begin{matrix}
\sqrt{3x^2+6x+7}=\sqrt{3(x+1)^2+4}\geq 2 & \\
 \sqrt{5x^2+10x+14}=\sqrt{5(x+1)^2+9}\geq 3&
\end{matrix}\right.
\Rightarrow \sqrt{3x^2+6x+7}+\sqrt{5x^2+10x+14}\geq 5$$

mặt khác: $$4-2x-x^2=5-(x+1)^2\leq 5$$

từ đây dễ dàng suy ra nghiệm $$x=-1$$



#14
Kaito Kuroba

Kaito Kuroba

    Thiếu úy

  • Thành viên
  • 656 Bài viết

15.$\sqrt{2-x^{2}}+\sqrt{2-\frac{1}{x^{2}}}=4-(x+\frac{1}{x})$

15.

ta có: $$\left\{\begin{matrix}
x+\sqrt{2-x^2}\leq 2 & \\
 \frac{1}{x}+\sqrt{2-\frac{1}{x^2}}\leq 2&
\end{matrix}\right.
\Rightarrow x+\sqrt{2-x^2}+\frac{1}{x}+\sqrt{2-\frac{1}{x^2}}\leq 4$$



#15
Kaito Kuroba

Kaito Kuroba

    Thiếu úy

  • Thành viên
  • 656 Bài viết

16.$\frac{2\sqrt{2}}{\sqrt{x+1}}+\sqrt{x}=\sqrt{x+9}$

16.

đề phải là thế này mới đúng: $\frac{2\sqrt{2}}{\sqrt{x+1}}+\sqrt{x}=\sqrt{x+9}$

ta có: $$\frac{2\sqrt{2}}{\sqrt{x+1}}+\sqrt{x}\leq \left ( \frac{1}{x+1}+\frac{x}{x+1} \right )\left ( x+9 \right )=x+9$$

từ đây dễ dàng suy ra nghiệm: $x=\frac{1}{7}$



#16
Kaito Kuroba

Kaito Kuroba

    Thiếu úy

  • Thành viên
  • 656 Bài viết

18.$2x^{2}-11x+21-3\sqrt[3]{4x-4}=0$

18.

ta có: $$3\sqrt{4x-4}\leq 3.\frac{1}{4}.\frac{4x+12}{3}=x+3$$

ta cần chứng minh rằng: $$2x^{2}-11x+21\geq x+3\Leftrightarrow 2(x-3)^2\geq 0$$



#17
Kaito Kuroba

Kaito Kuroba

    Thiếu úy

  • Thành viên
  • 656 Bài viết

19.$x\sqrt{y-1}+2y\sqrt{x-1}=\frac{3xy}{2}$

19.

áp dụng AM-GM ta có:

$$\left\{\begin{matrix}
x\sqrt{y-1}\leq \frac{xy}{2} & \\
 2y\sqrt{x-1}\leq xy&
\end{matrix}\right.
\Rightarrow VT\leq VP$$

$$"="\Leftrightarrow x=y=2$$



#18
Kaito Kuroba

Kaito Kuroba

    Thiếu úy

  • Thành viên
  • 656 Bài viết

20.$\sqrt{(x+2)(2x-1)}-3\sqrt{x+6}=4-\sqrt{(x+6)(2x-1)}+3\sqrt{x+2}$

 

20.

pttd:$$\left (\sqrt{x+2}+\sqrt{x+6}  \right )\left ( \sqrt{2x-1}-3 \right )\leq 4
\Rightarrow \frac{\sqrt{2x-1}-3}{\sqrt{x+6}-\sqrt{x+2}}\leq 1
\Leftrightarrow \sqrt{2x-1}-3\leq \sqrt{x+6}-\sqrt{x+2}\Rightarrow x=7$$

 

P/s: bài này dùng BDT để chứng minh thì chưa ra :biggrin: :biggrin:


Bài viết đã được chỉnh sửa nội dung bởi Kaito Kuroba: 02-05-2014 - 16:19


#19
Kaito Kuroba

Kaito Kuroba

    Thiếu úy

  • Thành viên
  • 656 Bài viết

14.$\frac{x-3x^{2}}{2}+\sqrt{2x^{4}-x^{3}+7x^{2}-3x+3}=2$

14.

ta có:

$$VT=\frac{x-3x^2}{2}+\sqrt{(x^2+3)(2x^2-x+1)}\leq \frac{x-3x^2}{2}+\frac{3x^2-x+4}{2}=2=VP$$

từ đây ta dễ dàng suy ra pt có 2 nghiệm:$$\begin{bmatrix}
x=-1 & \\
 x=2&
\end{bmatrix}$$

 

 bài13 giống bài 9

 


Bài viết đã được chỉnh sửa nội dung bởi Kaito Kuroba: 02-05-2014 - 16:28


#20
Yagami Raito

Yagami Raito

    Master Tetsuya

  • Thành viên
  • 1333 Bài viết

 Bài tập 

Giải các Phương trình sau:

 

 

3.              $\dfrac{2005}{x+y}+\dfrac{x}{y+2004}+\dfrac{y}{4009}+\dfrac{2004}{2005+x}=\dfrac{2}{z}$

 

 

3.Với $a,b>0$ thì $(a+b)^2 \geq 4ab \Rightarow \frac{1}{ab} \geq \dfrac{4}{(a+b)^2}$ Do $z \in \mathbb{N*} \Rightarrow \dfrac{2}{z} \geq 2$ 

Dấu "=" xảy ra khi $z=1$

Ta có $VT=(\dfrac{2005}{x+y}+\dfrac{y}{4009})+(\dfrac{x}{y+2004}+\dfrac{2004}{2005+x})$

$=\dfrac{2005.4009+xy+y^2}{4009(x+y)}+\dfrac{x^2+2005x+2004y+2004^2}{(y+2004)(2005+x)}$

$\geq \dfrac{4(2005.2009+xy+y^2)}{(x+y+4009)^2}+\dfrac{4(x^2+2005x+2004y+2004^2)}{(x+y+4009)^2}$

$=2(\dfrac{(x+y+4009)^2+(x-2004)^2+(y-2005)^2}{(x+y+4009)^2}) \geq 2$

 

Dấu bằng xảy ra khi $x=2004;y=2005;z=1$


Bài viết đã được chỉnh sửa nội dung bởi Yagami Raito: 02-05-2014 - 17:04

:nav: Học gõ công thức toán học tại đây

:nav: Hướng dẫn đặt tiêu đề tại đây

:nav: Hướng dẫn Vẽ hình trên diễn đàn toán tại đây

--------------------------------------------------------------

 





2 người đang xem chủ đề

0 thành viên, 2 khách, 0 thành viên ẩn danh